Stay ahead of learning milestones! Enroll in a class over the summer!

G
Topic
First Poster
Last Poster
k a April Highlights and 2025 AoPS Online Class Information
jlacosta   0
Wednesday at 3:18 PM
Spring is in full swing and summer is right around the corner, what are your plans? At AoPS Online our schedule has new classes starting now through July, so be sure to keep your skills sharp and be prepared for the Fall school year! Check out the schedule of upcoming classes below.

WOOT early bird pricing is in effect, don’t miss out! If you took MathWOOT Level 2 last year, no worries, it is all new problems this year! Our Worldwide Online Olympiad Training program is for high school level competitors. AoPS designed these courses to help our top students get the deep focus they need to succeed in their specific competition goals. Check out the details at this link for all our WOOT programs in math, computer science, chemistry, and physics.

Looking for summer camps in math and language arts? Be sure to check out the video-based summer camps offered at the Virtual Campus that are 2- to 4-weeks in duration. There are middle and high school competition math camps as well as Math Beasts camps that review key topics coupled with fun explorations covering areas such as graph theory (Math Beasts Camp 6), cryptography (Math Beasts Camp 7-8), and topology (Math Beasts Camp 8-9)!

Be sure to mark your calendars for the following events:
[list][*]April 3rd (Webinar), 4pm PT/7:00pm ET, Learning with AoPS: Perspectives from a Parent, Math Camp Instructor, and University Professor
[*]April 8th (Math Jam), 4:30pm PT/7:30pm ET, 2025 MATHCOUNTS State Discussion
April 9th (Webinar), 4:00pm PT/7:00pm ET, Learn about Video-based Summer Camps at the Virtual Campus
[*]April 10th (Math Jam), 4:30pm PT/7:30pm ET, 2025 MathILy and MathILy-Er Math Jam: Multibackwards Numbers
[*]April 22nd (Webinar), 4:00pm PT/7:00pm ET, Competitive Programming at AoPS (USACO).[/list]
Our full course list for upcoming classes is below:
All classes run 7:30pm-8:45pm ET/4:30pm - 5:45pm PT unless otherwise noted.

Introductory: Grades 5-10

Prealgebra 1 Self-Paced

Prealgebra 1
Sunday, Apr 13 - Aug 10
Tuesday, May 13 - Aug 26
Thursday, May 29 - Sep 11
Sunday, Jun 15 - Oct 12
Monday, Jun 30 - Oct 20
Wednesday, Jul 16 - Oct 29

Prealgebra 2 Self-Paced

Prealgebra 2
Sunday, Apr 13 - Aug 10
Wednesday, May 7 - Aug 20
Monday, Jun 2 - Sep 22
Sunday, Jun 29 - Oct 26
Friday, Jul 25 - Nov 21

Introduction to Algebra A Self-Paced

Introduction to Algebra A
Monday, Apr 7 - Jul 28
Sunday, May 11 - Sep 14 (1:00 - 2:30 pm ET/10:00 - 11:30 am PT)
Wednesday, May 14 - Aug 27
Friday, May 30 - Sep 26
Monday, Jun 2 - Sep 22
Sunday, Jun 15 - Oct 12
Thursday, Jun 26 - Oct 9
Tuesday, Jul 15 - Oct 28

Introduction to Counting & Probability Self-Paced

Introduction to Counting & Probability
Wednesday, Apr 16 - Jul 2
Thursday, May 15 - Jul 31
Sunday, Jun 1 - Aug 24
Thursday, Jun 12 - Aug 28
Wednesday, Jul 9 - Sep 24
Sunday, Jul 27 - Oct 19

Introduction to Number Theory
Thursday, Apr 17 - Jul 3
Friday, May 9 - Aug 1
Wednesday, May 21 - Aug 6
Monday, Jun 9 - Aug 25
Sunday, Jun 15 - Sep 14
Tuesday, Jul 15 - Sep 30

Introduction to Algebra B Self-Paced

Introduction to Algebra B
Wednesday, Apr 16 - Jul 30
Tuesday, May 6 - Aug 19
Wednesday, Jun 4 - Sep 17
Sunday, Jun 22 - Oct 19
Friday, Jul 18 - Nov 14

Introduction to Geometry
Wednesday, Apr 23 - Oct 1
Sunday, May 11 - Nov 9
Tuesday, May 20 - Oct 28
Monday, Jun 16 - Dec 8
Friday, Jun 20 - Jan 9
Sunday, Jun 29 - Jan 11
Monday, Jul 14 - Jan 19

Intermediate: Grades 8-12

Intermediate Algebra
Monday, Apr 21 - Oct 13
Sunday, Jun 1 - Nov 23
Tuesday, Jun 10 - Nov 18
Wednesday, Jun 25 - Dec 10
Sunday, Jul 13 - Jan 18
Thursday, Jul 24 - Jan 22

Intermediate Counting & Probability
Wednesday, May 21 - Sep 17
Sunday, Jun 22 - Nov 2

Intermediate Number Theory
Friday, Apr 11 - Jun 27
Sunday, Jun 1 - Aug 24
Wednesday, Jun 18 - Sep 3

Precalculus
Wednesday, Apr 9 - Sep 3
Friday, May 16 - Oct 24
Sunday, Jun 1 - Nov 9
Monday, Jun 30 - Dec 8

Advanced: Grades 9-12

Olympiad Geometry
Tuesday, Jun 10 - Aug 26

Calculus
Tuesday, May 27 - Nov 11
Wednesday, Jun 25 - Dec 17

Group Theory
Thursday, Jun 12 - Sep 11

Contest Preparation: Grades 6-12

MATHCOUNTS/AMC 8 Basics
Wednesday, Apr 16 - Jul 2
Friday, May 23 - Aug 15
Monday, Jun 2 - Aug 18
Thursday, Jun 12 - Aug 28
Sunday, Jun 22 - Sep 21
Tues & Thurs, Jul 8 - Aug 14 (meets twice a week!)

MATHCOUNTS/AMC 8 Advanced
Friday, Apr 11 - Jun 27
Sunday, May 11 - Aug 10
Tuesday, May 27 - Aug 12
Wednesday, Jun 11 - Aug 27
Sunday, Jun 22 - Sep 21
Tues & Thurs, Jul 8 - Aug 14 (meets twice a week!)

AMC 10 Problem Series
Friday, May 9 - Aug 1
Sunday, Jun 1 - Aug 24
Thursday, Jun 12 - Aug 28
Tuesday, Jun 17 - Sep 2
Sunday, Jun 22 - Sep 21 (1:00 - 2:30 pm ET/10:00 - 11:30 am PT)
Monday, Jun 23 - Sep 15
Tues & Thurs, Jul 8 - Aug 14 (meets twice a week!)

AMC 10 Final Fives
Sunday, May 11 - Jun 8
Tuesday, May 27 - Jun 17
Monday, Jun 30 - Jul 21

AMC 12 Problem Series
Tuesday, May 27 - Aug 12
Thursday, Jun 12 - Aug 28
Sunday, Jun 22 - Sep 21
Wednesday, Aug 6 - Oct 22

AMC 12 Final Fives
Sunday, May 18 - Jun 15

F=ma Problem Series
Wednesday, Jun 11 - Aug 27

WOOT Programs
Visit the pages linked for full schedule details for each of these programs!


MathWOOT Level 1
MathWOOT Level 2
ChemWOOT
CodeWOOT
PhysicsWOOT

Programming

Introduction to Programming with Python
Thursday, May 22 - Aug 7
Sunday, Jun 15 - Sep 14 (1:00 - 2:30 pm ET/10:00 - 11:30 am PT)
Tuesday, Jun 17 - Sep 2
Monday, Jun 30 - Sep 22

Intermediate Programming with Python
Sunday, Jun 1 - Aug 24
Monday, Jun 30 - Sep 22

USACO Bronze Problem Series
Tuesday, May 13 - Jul 29
Sunday, Jun 22 - Sep 1

Physics

Introduction to Physics
Wednesday, May 21 - Aug 6
Sunday, Jun 15 - Sep 14
Monday, Jun 23 - Sep 15

Physics 1: Mechanics
Thursday, May 22 - Oct 30
Monday, Jun 23 - Dec 15

Relativity
Sat & Sun, Apr 26 - Apr 27 (4:00 - 7:00 pm ET/1:00 - 4:00pm PT)
Mon, Tue, Wed & Thurs, Jun 23 - Jun 26 (meets every day of the week!)
0 replies
jlacosta
Wednesday at 3:18 PM
0 replies
k i Adding contests to the Contest Collections
dcouchman   1
N Apr 5, 2023 by v_Enhance
Want to help AoPS remain a valuable Olympiad resource? Help us add contests to AoPS's Contest Collections.

Find instructions and a list of contests to add here: https://artofproblemsolving.com/community/c40244h1064480_contests_to_add
1 reply
dcouchman
Sep 9, 2019
v_Enhance
Apr 5, 2023
k i Zero tolerance
ZetaX   49
N May 4, 2019 by NoDealsHere
Source: Use your common sense! (enough is enough)
Some users don't want to learn, some other simply ignore advises.
But please follow the following guideline:


To make it short: ALWAYS USE YOUR COMMON SENSE IF POSTING!
If you don't have common sense, don't post.


More specifically:

For new threads:


a) Good, meaningful title:
The title has to say what the problem is about in best way possible.
If that title occured already, it's definitely bad. And contest names aren't good either.
That's in fact a requirement for being able to search old problems.

Examples:
Bad titles:
- "Hard"/"Medium"/"Easy" (if you find it so cool how hard/easy it is, tell it in the post and use a title that tells us the problem)
- "Number Theory" (hey guy, guess why this forum's named that way¿ and is it the only such problem on earth¿)
- "Fibonacci" (there are millions of Fibonacci problems out there, all posted and named the same...)
- "Chinese TST 2003" (does this say anything about the problem¿)
Good titles:
- "On divisors of a³+2b³+4c³-6abc"
- "Number of solutions to x²+y²=6z²"
- "Fibonacci numbers are never squares"


b) Use search function:
Before posting a "new" problem spend at least two, better five, minutes to look if this problem was posted before. If it was, don't repost it. If you have anything important to say on topic, post it in one of the older threads.
If the thread is locked cause of this, use search function.

Update (by Amir Hossein). The best way to search for two keywords in AoPS is to input
[code]+"first keyword" +"second keyword"[/code]
so that any post containing both strings "first word" and "second form".


c) Good problem statement:
Some recent really bad post was:
[quote]$lim_{n\to 1}^{+\infty}\frac{1}{n}-lnn$[/quote]
It contains no question and no answer.
If you do this, too, you are on the best way to get your thread deleted. Write everything clearly, define where your variables come from (and define the "natural" numbers if used). Additionally read your post at least twice before submitting. After you sent it, read it again and use the Edit-Button if necessary to correct errors.


For answers to already existing threads:


d) Of any interest and with content:
Don't post things that are more trivial than completely obvious. For example, if the question is to solve $x^{3}+y^{3}=z^{3}$, do not answer with "$x=y=z=0$ is a solution" only. Either you post any kind of proof or at least something unexpected (like "$x=1337, y=481, z=42$ is the smallest solution). Someone that does not see that $x=y=z=0$ is a solution of the above without your post is completely wrong here, this is an IMO-level forum.
Similar, posting "I have solved this problem" but not posting anything else is not welcome; it even looks that you just want to show off what a genius you are.

e) Well written and checked answers:
Like c) for new threads, check your solutions at least twice for mistakes. And after sending, read it again and use the Edit-Button if necessary to correct errors.



To repeat it: ALWAYS USE YOUR COMMON SENSE IF POSTING!


Everything definitely out of range of common sense will be locked or deleted (exept for new users having less than about 42 posts, they are newbies and need/get some time to learn).

The above rules will be applied from next monday (5. march of 2007).
Feel free to discuss on this here.
49 replies
ZetaX
Feb 27, 2007
NoDealsHere
May 4, 2019
series and factorials?
jenishmalla   6
N 11 minutes ago by Maximilian113
Source: 2025 Nepal ptst p4 of 4
Find all pairs of positive integers \( n \) and \( x \) such that
\[
1^n + 2^n + 3^n + \cdots + n^n = x!
\]
(Petko Lazarov, Bulgaria)
6 replies
jenishmalla
Mar 15, 2025
Maximilian113
11 minutes ago
A board with crosses that we color
nAalniaOMliO   1
N an hour ago by EmersonSoriano
Source: Belarusian National Olympiad 2025
In some cells of the table $2025 \times 2025$ crosses are placed. A set of 2025 cells we will call balanced if no two of them are in the same row or column. It is known that any balanced set has at least $k$ crosses.
Find the minimal $k$ for which it is always possible to color crosses in two colors such that any balanced set has crosses of both colors.
1 reply
nAalniaOMliO
Mar 28, 2025
EmersonSoriano
an hour ago
Inspired by JK1603JK
sqing   1
N an hour ago by Soupboy0
Source: Own
Let $ a,b,c\geq 0 $ and $ab+bc+ca=1.$ Prove that$$\frac{abc-2}{abc-1}\ge \frac{4(a^2b+b^2c+c^2a)}{a^3b+b^3c+c^3a+1} $$$$\frac{abc-1}{abc-2}\ge \frac{(\sqrt 2-1)(a^2b+b^2c+c^2a+1)}{a^3b+b^3c+c^3a+1} $$
1 reply
sqing
an hour ago
Soupboy0
an hour ago
Olympiad Geometry problem-second time posting
kjhgyuio   7
N an hour ago by kjhgyuio
Source: smo problem
In trapezium ABCD,AD is parallel to BC and points E and F are midpoints of AB and DC respectively. If
Area of AEFD/Area of EBCF =√3 + 1/3-√3 and the area of triangle ABD is √3 .find the area of trapezium ABCD
7 replies
kjhgyuio
Apr 2, 2025
kjhgyuio
an hour ago
Geometry problem-second time posting
kjhgyuio   0
an hour ago
Source: smo roudn 2

A square is cut into several rectangles, none of which is a square ,so that the sides of each rectangles are parallel to the sides of a square .For each rectangle with sides a,b,a<b compute the ratio a/b Prove that the sum of these ratios is at least 1
0 replies
kjhgyuio
an hour ago
0 replies
Proving ZA=ZB
nAalniaOMliO   5
N an hour ago by EmersonSoriano
Source: Belarusian National Olympiad 2025
Point $H$ is the foot of the altitude from $A$ of triangle $ABC$. On the lines $AB$ and $AC$ points $X$ and $Y$ are marked such that the circumcircles of triangles $BXH$ and $CYH$ are tangent, call this circles $w_B$ and $w_C$ respectively. Tangent lines to circles $w_B$ and $w_C$ at $X$ and $Y$ intersect at $Z$.
Prove that $ZA=ZH$.
Vadzim Kamianetski
5 replies
nAalniaOMliO
Mar 28, 2025
EmersonSoriano
an hour ago
Inequality from China
sqing   3
N an hour ago by sqing
Source: lemondian(https://kuing.cjhb.site/thread-13667-1-1.html)
Let $x\in (0,\frac{\pi}{2}) . $ Prove that $$tanx\ge x^k$$Where $ k=1,2,3,4.$
3 replies
sqing
Yesterday at 1:11 PM
sqing
an hour ago
NMO (Nepal) Problem 4
khan.academy   8
N 2 hours ago by godchunguus
Find all integer/s $n$ such that $\displaystyle{\frac{5^n-1}{3}}$ is a prime or a perfect square of an integer.

Proposed by Prajit Adhikari, Nepal
8 replies
khan.academy
Mar 17, 2024
godchunguus
2 hours ago
2019 Nepal National Mathematics Olympiad
Piinfinity   3
N 2 hours ago by godchunguus
Problem 31
Let $f:\mathbb{R}\to\mathbb{R}$ be a function such that
$f(f(x))=x^2-x+1$
for all real numbers $x$. Determine $f(0)$.
3 replies
Piinfinity
Oct 13, 2020
godchunguus
2 hours ago
pretty well known
dotscom26   2
N 3 hours ago by Giant_PT
Let $\triangle ABC$ be a scalene triangle such that $\Omega$ is its incircle. $AB$ is tangent to $\Omega$ at $D$. A point $E$ ($E \notin \Omega$) is located on $BC$.

Let $\omega_1$, $\omega_2$, and $\omega_3$ be the incircles of the triangles $BED$, $ADE$, and $AEC$, respectively.

Show that the common tangent to $\omega_1$ and $\omega_3$ is also tangent to $\omega_2$.

2 replies
dotscom26
Yesterday at 2:03 AM
Giant_PT
3 hours ago
Unsolved NT, 3rd time posting
GreekIdiot   10
N 3 hours ago by mathprodigy2011
Source: own
Solve $5^x-2^y=z^3$ where $x,y,z \in \mathbb Z$
Hint
10 replies
GreekIdiot
Mar 26, 2025
mathprodigy2011
3 hours ago
Wot n' Minimization
y-is-the-best-_   24
N 3 hours ago by maromex
Source: IMO SL 2019 A3
Let $n \geqslant 3$ be a positive integer and let $\left(a_{1}, a_{2}, \ldots, a_{n}\right)$ be a strictly increasing sequence of $n$ positive real numbers with sum equal to 2. Let $X$ be a subset of $\{1,2, \ldots, n\}$ such that the value of
\[
\left|1-\sum_{i \in X} a_{i}\right|
\]is minimised. Prove that there exists a strictly increasing sequence of $n$ positive real numbers $\left(b_{1}, b_{2}, \ldots, b_{n}\right)$ with sum equal to 2 such that
\[
\sum_{i \in X} b_{i}=1.
\]
24 replies
1 viewing
y-is-the-best-_
Sep 23, 2020
maromex
3 hours ago
Functional equations
hanzo.ei   12
N 3 hours ago by truongphatt2668
Source: Greekldiot
Find all $f: \mathbb R_+ \rightarrow \mathbb R_+$ such that $f(xf(y)+f(x))=yf(x+yf(x)) \: \forall \: x,y \in \mathbb R_+$
12 replies
hanzo.ei
Mar 29, 2025
truongphatt2668
3 hours ago
Kosovo Mathematical Olympiad 2016 TST , Problem 1
dangerousliri   2
N 3 hours ago by navier3072
Solve equation in real numbers

$\sqrt{x+\sqrt{4x+\sqrt{16x+\sqrt{…+\sqrt{4^nx+3}}}}}-\sqrt{x}=1$
2 replies
dangerousliri
Jan 9, 2017
navier3072
3 hours ago
Acute triangle, its circumcenter and a point lies inside it
Fang-jh   35
N Feb 14, 2025 by Saucepan_man02
Source: USA TST 2005, Problem 6
Let $ABC$ be an acute scalene triangle with $O$ as its circumcenter. Point $P$ lies inside triangle $ABC$ with $\angle PAB = \angle PBC$ and $\angle PAC = \angle PCB$. Point $Q$ lies on line $BC$ with $QA = QP$. Prove that $\angle AQP = 2\angle OQB$.
35 replies
Fang-jh
Jan 21, 2007
Saucepan_man02
Feb 14, 2025
Acute triangle, its circumcenter and a point lies inside it
G H J
Source: USA TST 2005, Problem 6
The post below has been deleted. Click to close.
This post has been deleted. Click here to see post.
Fang-jh
237 posts
#1 • 7 Y
Y by Davi-8191, Adventure10, Rounak_iitr, ehuseyinyigit, and 3 other users
Let $ABC$ be an acute scalene triangle with $O$ as its circumcenter. Point $P$ lies inside triangle $ABC$ with $\angle PAB = \angle PBC$ and $\angle PAC = \angle PCB$. Point $Q$ lies on line $BC$ with $QA = QP$. Prove that $\angle AQP = 2\angle OQB$.
Z K Y
The post below has been deleted. Click to close.
This post has been deleted. Click here to see post.
Huyền Vũ
91 posts
#2 • 10 Y
Y by Abdollahpour, HolyMath, Asuna002, Adventure10, Mango247, and 5 other users
Solution:
(O2),(O1) are the excircles of triangle APB and APC
Because of ∠PAB=∠PBC and ∠PAC=∠PCB so BC is the common tangent of (O1) and (O2).
AP is the radical axis of (O1) and (O2) so AP intersecs BC at M where M is the midpoint of BC. So ∠OMQ=90 (1)
Triangle QAB and QCA are similar so QA^2=QB*QC.
Thus, QA is the tangent of the circle (O) or ∠OAQ=90 (2)
From (1) and (2) O,A,Q,M are concyclic
So ∠OMA=∠OQA
But we also have ∠OMA=∠MQO1 (Q,O1,O2 are collinear)
So ∠MQO1=∠AQO.
So ∠MQO=∠AQO1.
Final ∠AQP=2∠AQO1=2∠OQB.
Attachments:
Z K Y
The post below has been deleted. Click to close.
This post has been deleted. Click here to see post.
epitomy01
240 posts
#3 • 3 Y
Y by Adventure10, Mango247, and 1 other user
Here's another approach, slightly different to what Huyen did:
I'll use the diagram in the previous post.
$ Q, O_{2}, O_{1}$ are collinear since they all lie on the perpendicular bisector of $ AP$. If $ AP$ meets $ BC$ at $ A_{1}$, we have $ BPA_{1}$ and $ ABA_{1}$ are similar; also $ CPA_{1}$ and $ A_{1}CA$ are similar, so $ BA_{1}^2 = AA_{1} * PA_{1} = CA_{1}^2$, so $ BA_{1} = CA_{1}$.
Since $ QB, BO_{2}$ are perpendicular, and $ QC, CO_{1}$ are perpendicular, triangles $ QO_{2}B$ and $ QO_{1}C$ are similar, so $ \frac {O_{2} A}{O_{1} A} = \frac {O_{2}B}{O_{1}B} = \frac {QO_{2}}{QO_{1}}$. Considering triangle $ AO_{1}O_{2}$, and the point $ Q$ on $ O_{2} O_{1}$, by the converse of the external version of the Angle Bisector Theorem, $ QA$ is the external angle bisector of $ AO_{2}O_{1}$.
A quick angle chase gives: $ \angle O_{2}AB = 90 - B$, $ \angle OAB = 90 - C$, so $ \angle OAO_{2} = A$, and similarly $ \angle O_{1}AO = A$, so $ \angle O_{1}AO_{2} = 2A$, so since $ QA$ bisects the external angle $ O_{2}AO_{1}$ and $ AO$ bisects the internal angle $ O_{2}AO_{1}$, $ \angle QAO = 90$. Thus $ QA$ is tangent to the circumcircle of $ ABC$, and $ QAB$ is similar to $ QAC$. Thus $ QA^2 = QB * QC$ ... (1).
Since $ OO_{2}$ is perpendicular to $ AB$, $ OO_{2}$ bisects angle $ AOO_{2}$, so $ \angle AOO_{2} = C$, and since $ \angle OAO_{2} = A$, it follows that $ AO_{2}O$ is similar to $ ABC$. Similarly, $ AOO_{1}$ is similar to $ ABC$, and also $ AOO_{2}$. This gives us: $ AO^2 = AO_{2} * AO_{1}$ ... (2).
Combining (1) and (2), gives us:
$ (\frac {AQ}{AO})^2 = (\frac {QB}{O_{2}B})(\frac {QC}{O_{1}C})$ ... (3)
Since $ \frac {QB}{O_{2}B} = \frac {QC}{O_{1}C}$, since $ QBO_{2}$ and $ QCO_{1}$ are similar, we have
$ \frac {AQ}{AO} = \frac {QB}{O_{2}B}$.
Since $ \angle QBO_{2} = \angle QAO$, we have $ QBO_{2}$ and $ QAO$ are similar triangles, so $ \angle AQO = \angle O_{2}QB$, so $ \angle AQO_{2} = \angle OQB$, so $ \angle AQP = 2 \angle AQO_{2} = 2\angle OQB$, as required.

Some other properties of the point $ P$:
- The reflection of $ P$ about the midpoint $ A_{1}$ of $ BC$ lies on the circumcircle.
- If $ Q$ is the reflection $ P$ about $ BC$, $ Q$ lies on the circumcircle, and further, $ AQ$ is a symmedian of triangle $ ABC$.
- $ \frac {BP}{CP} = \frac {AB}{AC}$, so $ P$ lies on the $ A$- Apollonius circle of $ ABC$.
Z K Y
The post below has been deleted. Click to close.
This post has been deleted. Click here to see post.
greentreeroad
484 posts
#4 • 7 Y
Y by HDZ, Mediocrity, Awesome_guy, Adventure10, Mango247, and 2 other users
Another way:

suppose R is the reflection of P in BC, it's easy to see that A,B,R,C is cyclic.
So OA=OR.
since QA=QP=QR, so A,P,R lies on the circle center at Q with radius QA.
so <ARP=half of <AQP
since OA=OR, QA=QR, so OQ is perpendicular to AR, and since PR is perpendicular to QC, so it's easy to see <ARP=<OQB
thus <AQP=s<OQB
Z K Y
The post below has been deleted. Click to close.
This post has been deleted. Click here to see post.
The QuattoMaster 6000
1184 posts
#5 • 3 Y
Y by Adventure10, Mango247, and 1 other user
Here is another solution:
Solution
Z K Y
The post below has been deleted. Click to close.
This post has been deleted. Click here to see post.
leader
339 posts
#6 • 3 Y
Y by Adventure10, Mango247, and 1 other user
denote $S$ the midpoint of $AP$,$H$ orthocenter of $\triangle ABC$ ,$M=AP\cap BC$.$A'$ symmetric point of $A$ wrt $M$,. Since $\triangle MPB\sim \triangle MAB$ ie $MB^2=MA*MP$ similarly $MC^2=MP*MA$ so $MB=MC$ now diagonals of $ABA'C$ bisect each other so $ABA'Ç$ is a parallelogram. now $\angle A'CH=90$ but since $\angle APC=180-\angle PAB-\angle PAC=180-\angle ABC=\angle BHC=180-\angle BA'C$ we have $C,P,H,B,A'$ are con-cyclic. so $\angle HPA'=90$ since $QA=QP$ ie $SP=SA=AP/2$ and $\angle QSM=\angle QSP=90$ since $\angle PAH=\angle SQM$ and $\angle HPA=\angle QSM=90$ ie $\triangle QSM\sim \triangle AHP$ so $QS/QM=AP/AH$ since $AP=2*SP$ and $AH=2*OM$ ie $AP/AH=SP/OM$ but now $\angle QSP=\angle OMQ=90$ and $OM/SP=QM/QS$ so $\triangle OMQ\sim \triangle SPQ$ therefore $\angle SQP=\angle OQM=\angle OQB$ but since $AQ=QP$ ie $\angle AQP=2*\angle SQP$ so $\angle AQP=2*\angle OQB$
Z K Y
The post below has been deleted. Click to close.
This post has been deleted. Click here to see post.
thecmd999
2860 posts
#7 • 2 Y
Y by Adventure10, Mango247
Solution
Z K Y
The post below has been deleted. Click to close.
This post has been deleted. Click here to see post.
IDMasterz
1412 posts
#8 • 3 Y
Y by Adventure10, Mango247, and 1 other user
First, let use get the wording out of the way. Let $DEF$ be the orthic triangle. It is saying that $P$ is the intersection of $\odot AHEF$ with $BHC$. Now, we cant solve this straight away from here, we will need a few other stuff. Indeed, note that if $M$ is the midpoint of $BC$ that $EF$ is the polar of $M$, so it follows $\angle MFP = \angle FAP = \angle PBM$ (given) so the $P \in \odot MBF$ and do symmetry for other. Then, by radical axis, $PM \cap BF \cap BE = A$ so $P \in AM$. Now, we have enough so we ignore and focus on what we need to prove. We need $\angle QAM = \angle QOM \implies QAOM$ is cyclic. Let us prove $QA$ is tangent to $\odot ABC$. Since $Q$ is on the radical axis, it suffice to prove $QP$ is tangent to $\odot BHC$, and by symmetry this means if $AH \cap \odot BHC = A'$ then $QA'$ is tangent to $\odot BHC$. But, suppose the tangent at $P$ touches $BC$ at $Q'$. Then note $PBA'C$ is harmonic quad, so $Q'A'$ is tangent to $\odot BHC$ and by symmetry $Q'A = Q'P$ so $Q' = Q$ and we are done.
Z K Y
The post below has been deleted. Click to close.
This post has been deleted. Click here to see post.
fclvbfm934
759 posts
#9 • 3 Y
Y by Adventure10, Mango247, and 1 other user
Let $O_1$ and $O_2$ be the circumcircles of $ABP$ and $ACP$, respectively. Notice that $O_1O_2$ is the perpendicular bisector of $AP$, so $O_1O_2 \cap BC = Q$. Let $M = AP \cap BC$. Let $Q'$ be on $BC$ such that $Q'A$ is tangent to the circumcircle of $ABC$.

From $\angle PAB = \angle PBC$ and $\angle PAC = \angle PCB$, we know that the circumcircles of $ABP$ and $APC$ are both tangent to $BC$, and $AP$ is the radical axis, so that means $MB^2 = MC^2$, so $M$ is the midpoint of $BC$.

We now show that $Q' = Q$. We have:
\[\angle APB = 180^{\circ} - \angle BAP - \angle PBA = 180^{\circ} - \angle ABC \\ \angle APC = 180^{\circ} - \angle PAC - \angle PCA = 180^{\circ} - \angle BCA\]
From Law of Sines, we have $O_1B = \frac{AB}{2\sin{ABC}}$ and $O_2C = \frac{AC}{2\sin{ACB}}$. Therefore, $\frac{QB}{QC} = \frac{AB \sin{ACB}}{AC \sin{ABC}} = \frac{AB^2}{AC^2}$.

But since $Q'AB \sim Q'CA$, we have $\frac{Q'B}{Q'A} = \frac{AB}{AC}$ and $\frac{Q'A}{Q'C} = \frac{AB}{AC}$, and multiplying these two together yields $\frac{Q'B}{Q'C} = \frac{AB^2}{AC^2}$. Therefore, we have $Q = Q'$, so $QA$ is tangent to the circumcircle of $ABC$.

But that means $OA \perp QA$ and $OM \perp BC$, so $QAOM$ is cyclic, so we know that $QO$ passes through the circumcenter of $AQM$. But $QO_1 \perp AP$, so $QO$ is the isogonal conjugate of $QO_1$ with respect to $AQM$, so $\angle AQO_1 = \frac{1}{2}\angle AQP = \angle OQB$, and multiplying that equation by $2$ gives us the desired conclusion. $\blacksquare$
Z K Y
The post below has been deleted. Click to close.
This post has been deleted. Click here to see post.
sunken rock
4379 posts
#10 • 2 Y
Y by Adventure10, Mango247
The following solution is merely to prove the statements from the first given solution:
Let $O_b, O_c$ be the circumcenters of triangles $ABP, APC$ and $AD$ altitude of $\triangle ABC$.
We have $\angle BAO_b=\angle ABO_b=\angle BAD=\angle OAC\ (\ 1\ )$, $\angle CAO_b=\angle ACO_c=\angle DAC=\angle BAO\ (\ 2\ )$, from the above easily $\angle O_bAO=\angle DAO_c$, so $AO$ is angle bisector of $\angle O_bAO_c$; $O_bO_c$ being perpendicular bisector of $AP$, $Q, O_b, O_c$ are collinear, hence $\frac{CK}{BK}=\frac{CO_c}{BO_b}=\frac{AO_c}{AO_b}$, so $AK$ is external angle bisector of $\angle O_bAO_c$, i.e. $AQ\bot OA$, consequently $AOMQ$ is cyclic, of diameter $OQ$ and the altitude $QO_b$ of $\triangle AMQ$ is its isogonal w.r.t. $\angle MQA$, done.

Best regards,
sunken rock
Z K Y
The post below has been deleted. Click to close.
This post has been deleted. Click here to see post.
bojler
16 posts
#11 • 2 Y
Y by Adventure10, Mango247
Denote by $M$ the intersection of $AP$ and $BC$. We have $MB^2=MP*MA=MC^2$ so $M$ is midpoint of $BC$. Now let $P'$ be the reflection of $P$ in $BC$. $\angle BP'C=\angle BPC=180-\angle BAC$ so $P'$ is on circumcircle of triangle $ABC$.
$\angle BMP'=\angle BMA$ so quadrilateral $ABP'C$ is harmonic hence tangents at $A$ and $P'$ (on circumcircle of $ABC$) meet $BC$ at one point,let's call it $Q'$. $Q'P=Q'P'=Q'A$ so $Q'=Q$. Quadrilateral $QAOM$ is cyclic so $90-\frac{1}{2}\angle AQP=\angle QAM=\angle QOM=90-\angle OQM$ so $2\angle QOM=\angle AQP$.
Z K Y
The post below has been deleted. Click to close.
This post has been deleted. Click here to see post.
junioragd
314 posts
#12 • 3 Y
Y by Adventure10, Mango247, and 1 other user
Seems too easy,so it might be wrong.It is easy to see that $<BPC=<180-<BAC$,now let $O'$ be the reflection of $O$ wrt $BC$.Now,we have to prove $<OQO'=PQA$ and since we have $QA=QP$ and $QO=QO'$,so it is enough to prove that triangles QPO' and QOA are congruent,but since we have QP=QA and QO=QO',it is enough to prove that $O'P = AO$,but this is true since $O'$ is the circumcentre of $BPC$ and $O'P=O'B=O'C=OB=OC=AO$,so we are finished.
This post has been edited 1 time. Last edited by junioragd, Nov 5, 2014, 5:59 PM
Z K Y
The post below has been deleted. Click to close.
This post has been deleted. Click here to see post.
IDMasterz
1412 posts
#13 • 2 Y
Y by Adventure10, Mango247
junioragd wrote:
Seems too easy,so it might be wrong.It is easy to see that <BPC=<180-<BAC,now let O' be the reflection of O wrt BC.Now,we have to prove <OQO'=PQA and since we have QA=QP and QO=QO',so it is enough to prove that triangles QPO' and QOA are congruent,but since we have QP=QA and QO=QO',it is enough to prove that O'P = AO,but this is true since O' is the circumcentre of BPC and O'P=O'B=O'C=OB=OC=AO,so we are finished.

Yes it isn't very difficult, though it is very tempting to overkill it...

Looking back at the problem, it is very natural to reflect $P$ about $BC$ to make $P'$, radical axis gives $OQ \perp AP'$, $\angle AQP = 2\angle AP'P$ so we need $\angle AP'P = \angle OQB$ which is obvious as $AP' \perp OQ, P'P \perp QB$.
Z K Y
The post below has been deleted. Click to close.
This post has been deleted. Click here to see post.
Dukejukem
695 posts
#14 • 1 Y
Y by Adventure10
Let line $AP$ meet $BC$ at $M.$ Then since $\angle MAB = \angle PBM$, we deduce that $\triangle MAB \sim \triangle MBP.$ By writing the length ratios, it follows that \[MB^2 = MP \cdot MA.\] Similarly, we find that $\triangle MAC \sim \triangle MCP$, and $MC^2 = MP \cdot MA.$ Hence, $MB = MC$, so $M$ is the midpoint of $\overline{BC}.$ In addition, note that \[\angle BPC = 180^{\circ} - \angle PBC - \angle PCB = 180^{\circ} - \angle PAB - \angle PAC - 180^{\circ} - A.\] It follows that the reflection $P'$ of $P$ about $BC$ lies on $\odot (ABC).$

Now, notice from $\triangle MAB \sim \triangle MBP$, we have $PB / AB = PM / BM.$ Similarly, we obtain $PC / AC = PM / BM.$ Therefore, \[1 = \frac{PB}{AB} : \frac{PC}{AC} = \frac{P'B}{AB} : \frac{P'C}{AC}\] which implies that the quadrilateral $BACP'$ is harmonic. Then let the tangents to $\odot (ABC)$ at $A$ and $P'$ meet at $Q'.$ Note that since $BACP'$ is harmonic, we have $Q' \in BC.$ Combining equal tangents with the fact that $BC$ is the perpendicular bisector of $\overline{AA'}$ and $\overline{PP'}$, it follows that \[Q'A = Q'P' = Q'P.\] Hence, $Q' \equiv Q.$ It follows that $QA \perp OA$, so the points $Q, A, O, M$ are inscribed in a circle of diameter $\overline{QO}.$ Then $\angle QAP = \angle QOM$, which implies that $90^{\circ} - \frac{1}{2}\angle AQP = 90^{\circ} - \angle OQM$, whence the desired result follows immediately. $\square$
Z K Y
The post below has been deleted. Click to close.
This post has been deleted. Click here to see post.
Abdollahpour
63 posts
#15 • 1 Y
Y by Adventure10
This question must solve by the famous lemma.
First we must know what is the point "$P$" this point is absolutely important.This point is on the median of $BC$.
And we should know when we draw the circumcircle of the triangle $ABP$.that circumcircle tagents to $BC$.
This post has been edited 1 time. Last edited by Abdollahpour, Jul 4, 2017, 7:48 AM
Z K Y
The post below has been deleted. Click to close.
This post has been deleted. Click here to see post.
Abdollahpour
63 posts
#16 • 1 Y
Y by Adventure10
Huyền Vũ wrote:
Solution:
(O2),(O1) are the excircles of triangle APB and APC
Because of ∠PAB=∠PBC and ∠PAC=∠PCB so BC is the common tangent of (O1) and (O2).
AP is the radical axis of (O1) and (O2) so AP intersecs BC at M where M is the midpoint of BC. So ∠OMQ=90 (1)
Triangle QAB and QCA are similar so QA^2=QB*QC.
Thus, QA is the tangent of the circle (O) or ∠OAQ=90 (2)
From (1) and (2) O,A,Q,M are concyclic
So ∠OMA=∠OQA
But we also have ∠OMA=∠MQO1 (Q,O1,O2 are collinear)
So ∠MQO1=∠AQO.
So ∠MQO=∠AQO1.
Final ∠AQP=2∠AQO1=2∠OQB.

$Very Good$
Z K Y
The post below has been deleted. Click to close.
This post has been deleted. Click here to see post.
Blast_S1
356 posts
#17 • 2 Y
Y by karitoshi, Adventure10
Define $M$ as $\overline{AP}\cap\overline{BC}$, $N$ as the midpoint of $\overline{AP}$, and $\omega$ as the circle with diameter $\overline{BC}$. Clearly $\overline{QN}$ is the perpendicular bisector of $\overline{AP}$.
Lemma: $\overline{AQ}$ is the tangent to $(ABC)$ at $A$

Proof: From $\triangle MPB\sim\triangle MBA$ and $\triangle MPC\sim\triangle MCA$, we can immediately obtain that
$$MB^2=MP\cdot MA=MC^2,$$or that $M$ is the midpoint of $\overline{BC}$, and thus the center of $\omega$. Furthermore, this implies that $A$ is the image of inversion of $P$ wrt $\omega$, and thus $\overline{QN}$ is the radical axis of $\omega$ and the circle with radius $0$ at $A$. Now, since $\overline{BC}$ is the radical axis of $\omega$ and $(ABC)$, we must have that $Q$ is the radical center of $A$, $(ABC)$, and $\omega$, and so $Q$ lies on radical axis of $A$ and $(ABC)$, which is the desired tangent line.
Now, to conclude, we note that $\overline{OM}\perp\overline{BC}$ and $\overline{OA}\perp\overline{AQ}$, so $A$, $Q$, $M$, and $O$ are concyclic, which implies that
$$\angle AOQ=\angle AMQ=\angle NMQ.$$Combining this with
$$\angle OAQ=\angle MNA=90^\circ$$yields that
$$\frac{1}{2}\angle AQP=\angle NQA=\angle OQA-\angle OQN=\angle MQN-\angle OQN=\angle OQM=\angle OQB,$$and so $\angle AQP=2\angle OQB$. Q.E.D.
This post has been edited 2 times. Last edited by Blast_S1, Dec 25, 2017, 9:15 PM
Z K Y
The post below has been deleted. Click to close.
This post has been deleted. Click here to see post.
kalazealot
334 posts
#18 • 3 Y
Y by Adventure10, Mango247, amirhsz
generization: conclusion is still true if P is any point on circle (BCH), H is orthocenter of ABC.
Z K Y
The post below has been deleted. Click to close.
This post has been deleted. Click here to see post.
thegameisover
10 posts
#19 • 1 Y
Y by Adventure10
This solution is similar to what epitomy01 did, but perhaps shorter.

Let $M$ be the intersection point of lines $AP$ and $BC$, it can be easily verified that $\bigtriangleup BPM$ and $\bigtriangleup ABM$ are similar, so $BM^2 = AM.PM$, in the same way we can get $CM^2 = AM.PM$, so $BM=CM$, i.e. $M$ is the midpoint of $BC$.
Define $O_1$ and $O_2$ as the circumcenters of $\bigtriangleup ACP$ and $\bigtriangleup ABP$ respectively. From the fact that $Q, O_1$ and $O_2$ all lie on the perpendicular bisector of $AP$, then $Q, O_1$ and $O_2$ are collinear. Also note that $BO_2$ and $CO_1$ are perpendicular to $QC$ (by the fact that $\angle PBC = \angle PAB$ and $\angle PCB = \angle PAC$). Next, we have $\frac{O_{2}A}{O_{1}A} = \frac{O_{2}B}{O_{1}C} = \frac{QO_2}{QO_1} \Rightarrow QA$ is the external angle bisector of $\angle O_{2}AO_1$. Furthermore, we have $\angle BAO_2 = \angle ABO_2 = 90^\circ - B$ and in the same way, $\angle CAO_{2} = 90^\circ - C$. However, we know that $\angle BAO = 90^\circ - C = \angle CAO_1 \Rightarrow \angle OAO_1 = \angle BAC$, in the same way we get $\angle OAO_2 = \angle BAC$, so $AO$ is the internal angle bisector of $\angle O_{2}AO_1$, hence $\angle QAO = 90^\circ$, i.e. $QA$ is tangent to $(ABC)$.
Now, observe that $AQMO$ is cyclic since $\angle QAO = \angle OMQ = 90^\circ$. This implies that $\angle OQM = 90^\circ - \angle QOM = 90^\circ - \angle QAM = \angle AQO_2$. But since $AQ=QP$, then $\angle PQO_2 = \angle AQO_2 = \angle OQM \Rightarrow \angle QAP = 2\angle OQB$, as desired.
This post has been edited 3 times. Last edited by thegameisover, Dec 24, 2019, 3:14 PM
Z K Y
The post below has been deleted. Click to close.
This post has been deleted. Click here to see post.
Ali3085
214 posts
#20 • 1 Y
Y by Rexaria112
let $\angle AQP=2x, \angle OQB=y$ and $M$ is the midpoint of $BC$
it's well-known that $P$ is the $A-HM$ point

claim:$P$ is on the A-appolonain circle $\omega$
proof:
Do $\sqrt{bc}$ inversion then $(\omega)^*$ is the perpindacular bisector of $BC$ but $P^*$ is the intersection of tangents from $B,C$ to $(ABC)$
$\blacksquare$
thus $AQ$ is tangent to $(ABC)$
$\angle OMQ= \angle OAQ=90 \implies AOMQ$ is cyclic
then $90-x=\angle QAP=\angle QOM=90-y$
and we win :D
Z K Y
The post below has been deleted. Click to close.
This post has been deleted. Click here to see post.
Awesome_guy
862 posts
#21 • 1 Y
Y by Stargate-NT-enthusiast
Since no one posted this elegant solution...
https://cdn.discordapp.com/attachments/735962077608542389/740653832245215313/Screen_Shot_2020-08-05_at_3.33.37_PM.png
Let $P'$ be the reflection of $P$ over $\overline{BC}$. Let $R$ be the intersection of the tangents to $(ABC)$ at $B$ and $C$. Let $M$ be the intersection of lines $AP$ and $BC$. Since $P$ is the $A$-Humpty point of $\triangle ABC$, $M$ is the midpoint of $\overline{BC}$ and $\angle QMO = 90^{\circ}$. Thus $\angle AQP = 2\angle OQB$ if and only if $\angle QAP = \angle QAM = \angle QOM$. Thus we wish to prove $QAOM$ is cyclic $\implies\angle OAQ = 90^{\circ}\implies\overline{QA}$ is tangent to $(ABC)$.

Note that since $\angle BP'C=\angle BPC=180^{\circ}-\angle BAC$, $P'$ lies on $(ABC)$. Let $Q'$ be the intersection between the tangents to $(ABC)$ at $A$ and $P'$. Proving $Q'=Q$ thus finishes. Note that since $P$ lies on the $A$-Apollonius circle of $\triangle ABC$, $\tfrac{AB}{AC}=\tfrac{PB}{PC}=\tfrac{P'B}{P'C}$. Thus $ABP'C$ is harmonic, and $A$, $P'$, $R$ are collinear. Thus by La Hire's theorem, since $R$ lies on the polar of $Q'$, $Q'$ must lie on the polar of $R$ with respect to $(ABC)\implies Q'$ lies on line $BC$. Further, note that $Q'A=Q'P'=Q'P$, thus $Q'$ also lies on the perpendicular bisector of $\overline{AP}$. Thus $Q'=Q$, as desired. $\blacksquare$
This post has been edited 2 times. Last edited by Awesome_guy, Aug 5, 2020, 10:58 PM
Z K Y
The post below has been deleted. Click to close.
This post has been deleted. Click here to see post.
Plops
946 posts
#22
Y by
Claim: $QA$ is tangent to $\odot (ABC)$.
Proof: It is well known that $AP \cap BC=M$ the midpoint of $BC$. Consider an inversion with center $M$ and radius $MB$. This swaps $A$ and $P$, so $QN$ is the radical axis of the degenerate circle at $A$ and the circle with diameter $BC$.
$$QB \times QC=QA^2$$and the result follows.

Then since $\angle OMQ=\angle QAO=\angle QAN=\frac{\pi}{2}$

$$\angle AQN=\angle MAO=\angle OQM$$
and we are done.
Z K Y
The post below has been deleted. Click to close.
This post has been deleted. Click here to see post.
606234
80 posts
#23 • 1 Y
Y by KaiDaMagical336
Solution: We can easily see that $P$ is the $A$-humpty point of $\triangle{ABC}$ so $P$ lies on the $A$-median. Let $AP \cap BM = D$. By the problem statement, we need $90  - \frac{\angle{BQA}}{2} = 90 - \angle{OQB} \implies \angle{QAD} = \angle{DOQ} \implies ODAQ$ would be cyclic, so $\angle{OAD} = 90$, or $AQ$ is tangent to the circumcircle of $\triangle{ABC}$. Thus, this prompts us to use Phantom Points, i.e. take some point $R$ s.t $RA$ is tangent to the circumcircle of $\triangle{ABC}$ and try to prove that $RA = RP$. From the properties of the Humpty point, we have $\frac{AB}{AC} = \frac{PB}{PC}$ and we also know that $\frac{AB}{AC} = \frac{BR^2}{CR^2} \implies \frac{PB}{PC} = \frac{BR^2}{CR^2}$. Now we will show that this leads to the fact that $RP$ is tangent to the circumcircle of $\triangle{PBC}$. As a matter of fact, if we let the tangent from $P$ to the circumcircle of $\triangle{PBC}$ intersect $BC$ at some point $S$, then we have $\frac{PB}{PC} = \frac{BS^2}{CS^2} \implies \frac{BR^2}{CR^2} = \frac{BS^2}{CS^2} \implies R = S$. Finally, by POP, we have $RA^2  = RB*RC = RP^2 \implies RA = RP$, as claimed. $\square$
This post has been edited 2 times. Last edited by 606234, Dec 14, 2020, 8:46 PM
Z K Y
The post below has been deleted. Click to close.
This post has been deleted. Click here to see post.
laikhanhhoang_3011
637 posts
#24
Y by
$P$ is $A$-humpty point of $\triangle{ABC}$ so $P$ lies on circle $A-Appolonius$ with ratio $\frac{AB}{AC}$ (familiar result).
We know that the center of $A-Appolonius$ is the intersection of tangent at $A$ of $(ABC)$ and $BC$, so that's $Q$.
Let $M,N$ be midpoint of $BC, AP$. We have $\overline{A,N,P,M}$
We need $\measuredangle OQB =\frac{\measuredangle AQP}{2}=\measuredangle AQN\Leftrightarrow \bigtriangleup AQN \sim \bigtriangleup OQM $
But $\measuredangle OQB =\frac{\measuredangle AQP}{2}=\measuredangle AQN\Leftrightarrow \measuredangle QAM =\measuredangle QOM \Leftrightarrow QAOM$ is cyclic, which is exactly true. (Q.E.D)
Z K Y
The post below has been deleted. Click to close.
This post has been deleted. Click here to see post.
MathLuis
1471 posts
#25
Y by
Fang-jh wrote:
Let $ABC$ be an acute scalene triangle with $O$ as its circumcenter. Point $P$ lies inside triangle $ABC$ with $\angle PAB = \angle PBC$ and $\angle PAC = \angle PCB$. Point $Q$ lies on line $BC$ with $QA = QP$. Prove that $\angle AQP = 2\angle OQB$.

By the angle condition we can deduce that $P$ is the A-humpty point, let $M$ the midpoint of $BC$, let $A'$ the 2nd point in $(O)$ that lies on the A-symedian, let $D$ the midpoint of $AP$ and $D'$ the midpoint of $PA'$. Its known that $P$ and $A'$ are symetric w.r.t. $BC$ hence $D'$ lies on $BC$ and $PA'$ is perpendicular to $BC$ and $QA=QP=QA'$ meaning that $Q$ is the intersection of the tangents from $A$ and $A'$ to $(O)$, hence $AOMA'Q$ is cyclic, now let $QD$ meet $PD'$ at $L$, by angle chasing:
$$\angle QLA'=\angle AMQ=\angle QMA' \implies O,A,L,M,A',Q \; \text{concyclic}$$Now the problem is equivalent to show that $\angle AQD=\angle OQB$ but now since $AM \parallel LO$ we have that arcs $AL$ and $OM$ are equal in $(OALMA'Q)$ hence $\angle AQD=\angle OQB$ holds, thus we are done :D
This post has been edited 1 time. Last edited by MathLuis, Jun 20, 2022, 8:21 PM
Z K Y
The post below has been deleted. Click to close.
This post has been deleted. Click here to see post.
JAnatolGT_00
559 posts
#26
Y by
Let $M$ be the midpoint of $BC.$ Clearly $\odot (ABP),\odot (ACP)$ tangent to $BC,$ so $M\in AP$ and $Q$ is the exsimilicenter of these circles. Now from $|QA|^2=|QB|\cdot |QC|$ we get $Q\in \odot (AOM),$ and hence $$\angle AQP=180^\circ-2\angle QAP=180^\circ -2\angle QOM=2\angle OQB \text{ } \blacksquare$$
This post has been edited 1 time. Last edited by JAnatolGT_00, Jun 20, 2022, 9:36 PM
Z K Y
The post below has been deleted. Click to close.
This post has been deleted. Click here to see post.
ike.chen
1162 posts
#27
Y by
Headsolves for the win :-D.


Let $M$ be the midpoint of $BC$, ray $AM$ meet $(ABC)$ again at $R$, $T = BB \cap CC$, $K = AT \cap BC$, $N = \overline{AKT} \cap OQ$, and the projection of $A$ onto $BC$ be $X$.

The angle conditions imply $P$ is the $A$-Humpty point, so $P$ lies on $\overline{AMR}$. Now, a well-known lemma regarding the $A$-Apollonian Circle wrt $BC$ yields $Q = AA \cap BC$. Thus, $N$ is actually the $A$-Dumpty point, which means $BNOCT$ is cyclic with diameter $OT$.

Because $AK$ and $AR$ are isogonal in $\angle BAC$, angle chasing yields $ABK \sim ARC$. Thus, $$\angle OQB = 90^{\circ} - QOM = 90^{\circ} - \angle NOT = \angle NTO = \angle KAX$$$$= 90^{\circ} - \angle AKB = 90^{\circ} - \angle ACR = 90^{\circ} - \angle QAR = \frac{180^{\circ} - 2 \angle QAP}{2}$$$$= \frac{180^{\circ} - \angle QAP - \angle QPA}{2} = \frac{\angle AQP}{2}$$as desired. $\blacksquare$

In Retrospect: One can just notice $AOMQ$ is cyclic, implying $$2 \measuredangle OQB = 2 \measuredangle OAM = 2(90^{\circ} - \measuredangle MAQ) = 2 \measuredangle QAM = \measuredangle QAP + \measuredangle APQ = \measuredangle AQP$$which is much simpler.
This post has been edited 1 time. Last edited by ike.chen, Nov 26, 2023, 8:58 AM
Z K Y
The post below has been deleted. Click to close.
This post has been deleted. Click here to see post.
djmathman
7936 posts
#28
Y by
Hah, lengths.

Let $M$ be the midpoint of $\overline{BC}$; it's known/not too hard to prove $\triangle ABM\sim\triangle BPM$ and $\triangle ACM\sim\triangle CPM$. This means
\[
\frac{PB}{PC} = \frac{PB/PM}{PC/PM} = \frac{AB/BM}{AC/CM} = \frac{AB}{AC}.
\]Now define $Q_0:= AA\cap BC$. We claim that $Q_0A = Q_0P$. To prove this, define the function $f$ via
\[
f(X) = PX^2 - \operatorname{pow}_{\odot(ABC)}(X);
\]remark that $f$ is an affine map. Observe $f(B) = PB^2$, $f(C) = PC^2$, and
\[
\frac{BQ_0}{Q_0C} = \frac{AB^2}{AC^2} = \frac{f(B)}{f(C)}.
\]It follows that $f(Q_0) = 0$, i.e. $Q_0A = Q_0P$.

Thus $Q_0\equiv Q$, ergo $QAOM$ is cyclic. The desired equality follows by an angle chase.
This post has been edited 1 time. Last edited by djmathman, Jan 20, 2023, 8:07 PM
Z K Y
The post below has been deleted. Click to close.
This post has been deleted. Click here to see post.
shendrew7
793 posts
#29
Y by
From the angle conditions, it is clear that $P$ is the $A$-Humpty point. Since $A$ and $P$ both lie on the $A$-Apollonius circle, $Q$ must be the center. It is well known this circle is orthogonal to $(ABC)$, so $\angle QAO = 90$.

Denote $M = AP \cap BC$ as the midpoint of $BC$. We then have $MOAQ$ cyclic, and hence
\[\angle MAQ = \angle MOQ \implies \angle PAQ = 90 - \angle OQB \implies \angle APQ = 2 \angle OQB.~\blacksquare\]
Z K Y
The post below has been deleted. Click to close.
This post has been deleted. Click here to see post.
Akacool
14 posts
#30 • 1 Y
Y by Mutse
Lets name the circumcircle of $ABC$ as $\omega$. If we denote the intersection of tangents to the $\omega$ at $B$ and $C$ as $S$ , the midpoint of $BC$ as $M$, orthocenter as $H$, and the intersection of $\omega$ and $AS$ as $L$.
It's well know that $P$ is the A-Humpty point of triangle $ABC$ so $A$, $P$, $M$ are collinear and $HP$ will be perpendicular to $AM$.
Lets take $Q$ as the intersection of the tangent at $A$ wrt $\omega$ with $BC$. Then $Q$ lies on $BC$ which implies $S$ lies on the polar of $Q$ wrt $\omega$ which implies that $QL$ is tangent to $\omega$. $AL$ is isogonal to $AM$ so that implies that $\angle PBC = \angle LBC$ and $\angle PCB = \angle LCB$ which leads to $L$ being the reflection of $P$ wrt $BC$ which in turn implies that $QP = QL = QA$. Thus if we take midpoint of $AP$ as $R$, $\angle ARQ = \angle OAQ = 90$ which makes $\angle OAM = \angle AQR$.
$\angle OMQ = \angle OAQ = 90$ so $AOMQ$ is cyclic implying $\angle OAM = \angle OQM = \angle AQR = \frac{1}{2}\angle AQP$ finishing the proof.
This post has been edited 4 times. Last edited by Akacool, Apr 25, 2024, 4:42 PM
Z K Y
The post below has been deleted. Click to close.
This post has been deleted. Click here to see post.
EthanWYX2009
842 posts
#31 • 1 Y
Y by ys-lg
Proof
Z K Y
The post below has been deleted. Click to close.
This post has been deleted. Click here to see post.
khanhnx
1618 posts
#32
Y by
Let $S$ be intersection of $A$ - symmedian of $\triangle ABC$ with $(O)$. From the hypothesis, we have $P$ is $A$ - Humpty point of $\triangle ABC$. So $Q$ must be the intersection of tangent at $A$ of $(O)$ with $BC$. Then $OQ \perp AS$. We have the familiar result that $PS \perp BC$. Hence $\angle{AQP} = 2\angle{ASP} = \angle{BQO}$
Z K Y
The post below has been deleted. Click to close.
This post has been deleted. Click here to see post.
amirhsz
16 posts
#33 • 2 Y
Y by Mahdi_Mashayekhi, ayeen_izady
If we add circumcenter $O'$ of $BHC$, then we have $O'P=O'B=OB$(because P lies on circumcircle of $BHC$ and $O'$ is reflected of $O$ with respect to $BC$). so we have $QO=QO', QA=QP, AO=O'P$ so $OAQ$ and $O'PQ$ are similar. So $QAP$ and $QOO'$ are similar too(why?) So we have $OQO'=2OQB=AOP$ and were done!!
Z K Y
The post below has been deleted. Click to close.
This post has been deleted. Click here to see post.
Aiden-1089
277 posts
#34
Y by
Note that $P$ is the $A$-Humpty point.
Take $\sqrt{bc}$ inversion, then $P$ goes to $T=BB \cap CC$. The image of $Q$ lies on the circle centred at $T$ passing through $A$, and on $(ABC)$, so it is the reflection of $A$ across the perpendicular bisector of $BC$. Call this point $A'$. Inverting back, we get that $Q=AA \cap BC$.
Let $M$ be the midpoint of $BC$, then since $\angle QAO = 90^\circ = \angle QMO$, $(AOMQ)$ is concyclic.
Let $H$ be the orthocentre, then
$$\frac{1}{2} \angle AQP = \frac{1}{2} \angle ATA' = \angle ATO = \angle TAH = \angle OAM = \angle OQM = \angle OQB$$so we are done. $\square$
Z K Y
The post below has been deleted. Click to close.
This post has been deleted. Click here to see post.
bin_sherlo
672 posts
#35
Y by
$P$ is $A-$humpty point. Perform $\sqrt{bc}$ inversion and reflect over the angle bisector of $\measuredangle CAB$.
New Problem Statement: $ABC$ is a triangle $(AB<AC)$ where $T$ is the intersection of tangents to $(ABC)$ at $B,C$ and $D\in (ABC)$ with $AD\parallel BC$. $A'$ is the reflection of $A$ over $BC$. Prove that $\measuredangle ATD=2\measuredangle B-2\measuredangle C-2\measuredangle AA'D$.
Let $M$ be the midpoint of $BC$ and $S$ be the intersection of $A-$symmedian with $(ABC)$. Since $\measuredangle DAA'=90$ and $MD=MA=MA'$ we see that $D,M,A'$ are collinear. Also
\[(BC_{\infty},M;B,C)=-1=(A,S;B,C)=(DA,DS;DB,DC)=(BC_{\infty},DS\cap BC;B,C)\]implies $D,M,S$ are collinear.
\[\frac{\measuredangle ATD}{2}+\measuredangle AA'D=\measuredangle ATM+\measuredangle TMA'=\measuredangle ASD=\measuredangle B-\measuredangle C\]As desired.$\blacksquare$
Z K Y
The post below has been deleted. Click to close.
This post has been deleted. Click here to see post.
Saucepan_man02
1300 posts
#36
Y by
Note that: $P$ is $A$ Humpty Point. Let $D, K$ denote the intersection of internal and external angle bisectors of $\angle A$ with $BC$ respectively. Note that: $Q$ is the center of the $A$ Apollonius circle, with diameter $DK$. Define couple of points:

-$Y=KA \cap (ABC)$.
-$M$ be the midpoint of $BC$.
-$X=BB \cap CC$
-$R=AX \cap (ABC)$ ($R \neq A$).
- $F$ b the midpoint of $AR$.
Note that: $Q, F, O$ are collinear as $OQ \perp AR$ ($A$ Apollonius circle is orthogonal to $(ABC)$). Also, note that $Y, O, M, X$ are collinear with $QFMX$ to be cyclic as $\angle QFX = \angle QMX = 90^\circ$.

By $\sqrt{bc}$ inversion at $A$:
-$P$ maps to $X$.
-$K$ maps to $Y$ .

Therefore: $\angle AKP = \angle AXY = \angle FXM$.
Notice that: $\angle OQB =\angle FQM = \angle FXM = \angle AKP = \frac{\angle AQD}{2}$.
Z K Y
N Quick Reply
G
H
=
a